¿Es el dirac lagrangiano hermitiano?

  1. Me pregunto por la densidad lagrangiana de Dirac.

    L = ψ ¯ ( i γ m m + metro ) ψ
    es un operador hermitiano, ya que al conjugar en complejo se obtiene
    L = ψ ( i γ 0 γ m γ 0 m + metro ) γ 0 ψ
    = ψ ¯ ( i γ m m + metro ) ψ L .

  2. ¿Y un lagrangiano debe ser siempre hermitiano? Sé que un operador hermitiano tiene valores propios reales, lo cual es deseable para un operador que describe observables. Pero aquí el Lagrangiano no es realmente un observable ya que se determina módulo una derivada total.

  3. Encontré una pregunta relacionada en el sitio: ¿dónde se dice que

    "la derivada m en el Dirac Lagrangian es antihermitiano" ( ¿Es real la densidad de Lagrangian en la teoría de campo? )

¿Puede alguien mostrarme cómo demostrar esto?

El Dirac Lagrangiano es hermitiano, pero no real. Ver aquí: physics.stackexchange.com/questions/529496/…

Respuestas (3)

( ψ γ 0 ψ ) = ψ γ 0 ψ
porque γ 0 es hermético. También,

( ψ i γ 0 γ m m ψ ) = i m ψ γ m γ 0 ψ = i m ψ ( γ 0 γ m γ 0 ) γ 0 ψ = i m ψ γ 0 γ m ψ = i ψ γ 0 γ m m ψ + s tu r F a C mi t mi r metro
Para la segunda línea usé γ m = γ 0 γ m γ 0 y para la última línea la integré por partes. Creo que su pregunta depende de esta parte, porque el último "índice" que resumimos es el índice del espacio-tiempo. X m , es decir, integración. Es la misma razón por la cual el operador de momento mecánico cuántico pag = i X es hermético.

Editar: Algo que pasé por alto es que los espinores también son números de Grassmann, por lo que se debe tener cuidado. En particular, esto significa que los componentes de los espinores satisfacen

( ψ i ϕ j ) = ϕ j ψ i

( más sobre eso aquí ). Ya se intercambian los objetos al tomar el conjugado hermitiano por las reglas del álgebra matricial, y existe la tentación de querer introducir un signo menos porque son números de Grassmann, pero esto sería redundante. Tomando prestado de la respuesta matemática.se vinculada :

( η ξ ) = [ ( a + i b ) ( C + i d ) ] = ( a C b d + i b C + i a d ) = C a d b i C b i d a = ( C i d ) ( a i b ) = ξ η

En el primer párrafo impones la condición de hermiticidad γ m = γ 0 γ m γ 0 ¿Eso no te cuesta la invariancia relativista?

En principio lo que importa es la acción y no la densidad lagrangiana que, en rigor, no debe ser considerada como un observable, ya que está definida hasta términos de frontera. Con respecto al Lagrangiano libre que está considerando, es real hasta un término límite. También puedes redefinirlo simplemente agregando el conjugado hermiteno de Lagrangiano que escribiste al original y tomando la mitad del resultado. Esta nueva densidad lagrangiana es real y equivalente a la inicial.

La densidad lagrangiana es real (siempre). Entonces, por definición, es hermitiano.

Esto podría ser mejor si proporcionara un poco más de exposición.
Esta respuesta es falsa. La densidad lagrangiana ni siquiera es siempre hermítica. Sin embargo, si no lo es, puede agregarle un término que no afecte la acción (o el EOM) que lo hace hermético. Sin embargo, esto no es solo por definición de lo que es una densidad lagrangiana. Tiene que ser mostrado para cada L individualmente.